<<

1 What was important about in Buddhist history? 1. It was ruled by Pasenadi who was a disciple of . 2. The famous Vihara is situated here. 3. It hosted the first sermon of the Buddha. Select the correct answer using the codes below. A. 1 only B. 1 and 2 only C. 2 and 3 only D. 1, 2 and 3

Correct Answer : B Answer Justification : Justification: Statement 1: About 2,500 years ago, it was one of the six largest cities of India and served as the capital of the Kingdom, and its king was called Pasenadi, who was a disciple of Buddha. The Buddha passed the greater part of his monastic life in Shravasti. Statement 2: It was also a prominent trading centre and center of religious activity by Buddha. According to the history of , The Buddha first came to Shravasti at the urging of Sudatta, a rich merchant who met the Buddha in Rajagir. Sudatta invited the Buddha to Shravasti and began to look for a suitable place to build a vihara. Q Source: Additional Research: 11th TN History Textbook 2 ‘SCORM’ was recently in news in the context of A. E-learning technical standards B. Recording information on DNA C. Hybrid crop technology D. Inter-operable airline standards

Correct Answer : A Answer Justification : Learning: SCORM is a set of technical standards for e-learning software products. SCORM tells programmers how to write their code so that it can “play well” with other elearning software. It is the de facto industry standard for e-learning interoperability. Specifically, SCORM governs how online learning content and Learning Management Systems (LMSs) communicate with each other (for e.g. similar standards for DVD players across countries). SCORM does not speak to instructional design or any other pedagogical concern — it is purely a technical standard. Q Source: Picked from the objectives of NMECIT http://mhrd.gov.in/sites/upload_files/mhrd/files/upload_document/MissionDocument.pdf 3 RBI has issued a Cyber Security Framework in Banks, mandating banks to 1. Conduct regular workshops for bank consumers with regard to cyber security awareness 2. Institute a board-approved cyber-security policy which covers the risks from cyber threats and the measures to address them 3. Reversal of erroneous debits arising from fraudulent or other transactions 4. Appoint a member of CERT-In as an integral part of the Board of Governors of all scheduled banks Select the correct answer using the codes below. A. 1, 3 and 4 only B. 2 and 3 only C. 1 and 2 only D. 2, 3 and 4 only

Correct Answer : B Answer Justification : Justification: Under the framework, RBI has issued instructions to banks for the following: Reversal of erroneous debits arising from fraudulent or other transactions Board-approved bank policy to cover customer protection Mechanism of compensating the customer for the unauthorised electronic banking transactions Display of the same on the bank’s website, along with the details of grievance-handling / escalation procedure. 4 Qualified Institutional Placement (QIP) allows 1. A listed company to issue equity shares for raising capital 2. Government to borrow from the market without causing any ‘crowding out effect’ Which of the above is/are correct? A. 1 only B. 2 only C. Both 1 and 2 D. None

Correct Answer : A Answer Justification : Concept: Crowding out effect happens when a large borrowing by the government leads to a shortage of capital for private players in the market. This leads to a rise in interest and thus increase in borrowing costs for the corporates. Justification: A QIP is a capital raising tool wherein a listed company can issue equity shares, fully and partly convertible debentures, or any security (other than warrants) that is convertible to equity shares. Apart from preferential allotment, this is the only other speedy method of private placement whereby a listed company can issue shares or convertible securities to a select group of investors. Statement 2: If there is government borrowing, a crowding out effect is most likely to follow because there isn’t enough capital for everyone in the market. How different from an IPO: But unlike in an IPO or an FPO (further public offer), only institutions or qualified institutional buyers (QIBs) can participate in a QIP issuance. QIBs include mutual funds, domestic financial institutions such as banks and insurance companies, venture capital funds, foreign institutional investors, and others. The State Bank of India is planning to complete a planned share sale by year-end, probably through a qualified institutional placement (QIP). 5 Tamil culture reached its peak under Cholas. Consider the following matches of major text works and their description. 1. Periya Puranam: Lives of the Nayanars 2. Kalingattupparani: Kalinga war fought by Kulotunga 3. Moovarula: Life of Chola kings 4. Kalladam: Work on Tamil grammar Select the correct answer using the codes below. A. 2 and 3 only B. 1, 2 and 3 only C. 3 and 4 only D. 1, 2, 3 and 4

Correct Answer : D Answer Justification : Justification: Statement 1: Sometimes called Tiruttontarpuranam, the Purana of the Holy Devotees, is a Tamil poetic account depicting the legendary lives of the sixty-three Nayanars, the canonical poets of Tamil Shaivism. Statement 2: Jayankondar’s Kalingattupparani describes the Kalinga war fought by Kulotunga I. Statement 3: Ottakoothar (another name) was a Tamil court poet to three legendary early Chola kings, namely Vikrama Chola, Kulotunga II and Rajaraja II. Statement 4: The works on Tamil grammar like Kalladam by Kalladanar, Yapperungalam by Amirthasagarar, a Jain, Nannul by Pavanandhi and Virasoliyam by Buddhamitra were the products of the Chola age. Q Source: Additional Research: 11th TN History Textbook 6 World's first sanctuary for White Tigers is located in which of these states of India? A. B. Assam C. Madhya Pradesh D. Uttar Pradesh

Correct Answer : C Justification: In India, white tigers are predominantly found in Rewa, Madhya Pradesh. In 2016, world’s first White Tiger Safari was inaugurated in Satna district of Madhya Pradesh. Learning: White tigers are found in other major zoos across India as well, for e.g. Nandankanan Zoological Park, Odisha. According to the scientists, white tigers lack pheomelanin, which is responsible for the red-yellow hue in the skin coat. 7 Consider the following about several major maritime choke points. 1. Strait of Mandeb connects the Red Sea to the Gulf of Aden. 2. Lombok Strait connects the Java Sea to the Indian Ocean. 3. Strait of Malacca connects North Atlantic Ocean to the South Pacific Ocean. Select the correct answer using the codes below. A. 1 and 2 only B. 2 and 3 only C. 3 only D. 1, 2 and 3 : Correct Answer : A Answer Justification : Justification: Statement 1: The Bab-el-Mandeb or Mandeb Strait is a strait located between Yemen on the Arabian Peninsula, and Djibouti and Eritrea in the Horn of Africa. It connects the Red Sea to the Gulf of Aden. Statement 2: The Lombok Strait, connects the Java Sea to the Indian Ocean, and is located between the islands of Bali and Lombok in Indonesia. The Gili Islands are on the Lombok side. Statement 3: From an economic and strategic perspective, the Strait of Malacca is one of the most important shipping lanes in the world. Q Source: Map based questions: Frequently seen in news The strait is the main shipping channel between the Indian Ocean and the Pacific Ocean, linking major Asian economies such as India, China, Japan, Taiwan, and South Korea. 8 India’s National Programme — Scientific Validation and Research on Panchagavya (SVAROP) — will be coordinated by the Department of Science and Technology (DST) and other agencies. Panchagavya is believed to be useful in A. Curing diseases like Arthritis, epilepsy and diabetes B. Treating oil spills C. Soaking polluted groundwater with heavy metals D. Clearing adulterated milk and urea on a large scale

Correct Answer : A Answer Justification : Learning: Panchagavya is a concoction of cow dung, cow urine, milk, curd and ghee. According to the TNAU website, the magic potion can “cure arthritis within two months” and help treat several other diseases and conditions such as epilepsy and Parkinson’s. It can reduce blood sugar levels and enable diabetic patients to reduce the dose of anti-diabetic drugs. A 19-member committee has been constituted to select projects that can help scientifically validate the benefits of panchagavya. Q Source: 9 Which of these are distinguishing features between Gandhara school of art and Mathura school of art? 1. Gandhara school originated during the reign of Kushanas whereas Mathura school originated under Sakas. 2. Gandhara school made sculptures of the Buddha, whereas Mathura school only portrayed traditional Indian Gods and Goddesses. Which of the above is/are correct? A. 1 only B. 2 only C. Both 1 and 2 D. None

Correct Answer : D Answer Justification : Justification: Statement 1: Gandhara originated during the reign of Indo-Greek rulers but the real patrons of this school of art were the Sakas and the Kushanas, particularly Kanishka. Mathura school was an indigenous development. Statement 2: Mathura school carved out the images of Buddha as well as Siva and Vishnu along with their consorts (Parvathi and Lakshmi). Gandhara art was a blend of Indian and Graeco-Roman elements. Its reliefs depict Buddha’s birth, his renunciation and his preaching. Q Source: 11th TN Textbook 10 Van Allen radiation belts would not have existed without A. Headley cells on higher latitudes formed on earth B. Magnetic field of earth C. Seasonal change in the equators D. Movement of cold currents in higher latitudes

Correct Answer : B Answer Justification : Concept: The Van Allen belts are a collection of charged particles, gathered in place by Earth’s magnetic field. Most of the particles that form the belts are thought to come from solar wind and other particles by cosmic rays. By trapping the solar wind, the magnetic field deflects those energetic particles and protects the Earth's atmosphere from destruction. Justification: Presumably the magnetic field of the Earth is generated by a dynamo effect that involves its rotation. If the Earth stopped rotating, it's magnetic field would no longer be regenerated and it would decay away to some low, residual value due to the very small component which is 'fossilized' in its iron-rich rocks. There would be no more 'northern lights' and the Van Allen radiation belts would probably vanish, as would our protection from cosmic rays and other high-energy particles. This is a significant biohazard. Q Source: https://www.nasa.gov/mission_pages/sunearth/news/gallery/ScienceCover.html 11 Peninsular plateaus in India are considered to be very useful because 1. It is rich in mineral deposits. 2. Its lava plateaus are generally rich in fertile black soil. 3. It is younger than most mountains in India and holds greater possibility for landscape transformation. 4. It hosts a global biodiversity hotspot. Select the correct answer using the codes below. A. 1 and 2 only B. 2, 3 and 4 only C. 1, 2 and 4 only D. 1 and 3 only

Correct Answer : C Answer Justification : Justification: Statement 1: It is store house of minerals like Iron ore, bauxite, gold, copper, coal, manganese, mica etc. Statement 2: Black soil of the Deccan plateau is known for growing cotton. Statement 3: Deccan plateau hosts the oldest rocks in India, so 3 is wrong. It is older than Himalayas. Statement 4: Western Ghats is located in Peninsular plateau. Q Source: 7th NCERT Geography 12 With reference to the Pradhan Mantri Matritva Vandana Yojana (PMMVY), consider the following: 1. It is a conditional cash transfer scheme for pregnant and lactating women and adolescent girls 2. It provides a partial wage compensation to women for wage-loss during childbirth and childcare Which of the above is/are correct? A. 1 only B. 2 only C. Both 1 and 2 D. None

Correct Answer : B Answer Justification : Justification: Previously Indira Gandhi Matritva Sahyog Yojana (IGMSY), is a maternity benefit program run by the government of India. Statement 1: It is only for pregnant and lactating women of 19 years of age or above for first two live births. Adolescents are legally not permitted to marry and bear children. Statement 2: This is for childbirth and childcare and to provide conditions for safe delivery and good nutrition and feeding practices. Q Source: 13 Rules laid down by the Vishnu Smriti for the grant of land provide for which of the following? 1. Landed property bestowed upon brahmanas by other rulers should not be appropriated by a King. 2. For the information of future rulers, land grants should be recorded containing the names of immediate ancestors. Which of the above is/are correct? A. 1 only B. 2 only C. Both 1 and 2 D. None

Correct Answer : C Answer Justification : Justification: This is the statement (translated) that appears in Vishnusmriti. “Let him (the king) bestow landed property upon brahmanas. To those upon whom he has bestowed (land), he must give a document, destined for the information of a future ruler, which must be written upon a piece of cloth, or a copper plate, or must contain the names of (three) immediate ancestors, a declaration of the extent of the land, and a description (of what results) from disrupting the donation and should be signed with his own seal. Let him not appropriate to himself landed property bestowed (upon brahmanas) by other rulers.” Q Source: Vishnu Smriti - The Archaeology of Hindu Ritual: http://vle.du.ac.in/mod/book/view.php?id=5353&chapterid=133614 Consider the following statement. By measuring the angle of the Pole Star from your place, you can know the ………….. of your place. 1. Latitude 2. Longitude 3. Altitude Select the correct answer using the codes below. A. 1 only B. 2 and 3 only C. 1 and 2 only D. 3 only

Correct Answer : A Answer Justification : Justification: Statement 1: If you were standing at the North Pole, the North Star would be directly above you. If you were standing at the Earth's equator, the North Star would be right along the horizon. If you measure the angle of the North Star above the horizon, that will be the same as your latitude. The North Pole is a latitude of 90 degrees north, and the equator is 0 degrees north. At the North Pole, the North Star is above, which is 90 degrees above the horizon. And, at the equator, the North Star is on the horizon, which is 0 degrees above the horizon. Statement 2: If you look at the diagram above, you will notice that you can’t decide your longitude because the angle measurements from any longitude would be the same. Q Source: 6th NCERT Geography 15 When blood is given to patients who are immuno-deficient such as cancer patients or patients being operated for organ transplants, blood needs to be irradiated with low doses of radiation. Why then high dosages of irradiation are not used to directly kill HIV viruses? 1. It would kill blood cells along with HIV viruses. 2. It can cause immediate T- Lymphocyte proliferation in blood cells. Which of the above is/are correct? A. 1 only B. 2 only C. Both 1 and 2 D. None

Correct Answer : A Answer Justification : Justification: Statement 1: Much higher dose of radiation will also kill the blood cells and will hence be useless. Hence this method cannot be resorted to for such a purpose. The same argument holds good for other bacteria and viruses also. This may lead to complications and may be fatal. Statement 2: T lymphocyte, is a type of lymphocyte that plays a central role in cell-mediated immunity. The irradiation is done in order to inhibit T- Lymphocyte proliferation so that the therapy doesn’t fail due to body’s internal defence mechanisms. Q Source: http://www.britatom.gov.in/htmldocs/faqs_ec2.html 16 The South Asia Growth Quadrangle (SAGQ) constitutes which of these countries? 1. Nepal 2. Bangladesh 3. Sri Lanka 4. India 5. Afghanistan Select the correct answer using the codes below. A. 1, 2 and 4 only B. 3 and 4 only C. 1, 2 and 3 only D. 1, 2, 3, 4 and 5

Correct Answer : A Answer Justification : Justification: SAGQ was launched in 1997 by the Foreign Ministers of Bangladesh, Bhutan, India, and Nepal (BBIN). SAARC in 1997 endorsed SAGQ as a sub-regional initiative under SAARC. The goals of SAGQ are: to create an enabling environment for accelerating economic growth; to overcome infrastructural constraints; to make optimal use of and further develop the complementarities in the subregion, and to improve policy coordination Learning: The South Asia Subregional Economic Cooperation (SASEC) Program, set up in 2001, brings together Bangladesh, Bhutan, India, Maldives, Myanmar, Nepal and Sri Lanka in a projectbased partnership to promote regional prosperity by improving cross-border connectivity, boosting trade among member countries, and strengthening regional economic cooperation. Q Source: Additional Research: http://pib.nic.in/newsite/PrintRelease.aspx?relid=160394 17 India has become the 71st nation to join the United Nations TIR (Transports Internationaux Routiers) Convention. How will this be helpful to India? 1. It will eliminate all border taxes, customs and duties between member countries. 2. It will enable India to move cargo along the International North-South Transport Corridor via Chabahar port in Iran. A. 1 only B. 2 only C. Both 1 and 2 D. None Your Answer : Correct Answer : B Answer Justification : Background: TIR Carnets, 1975 (TIR Convention) is an international customs transit system under the auspices of the United Nations Economic Commission for Europe (UNECE) with the widest geographical coverage. It is more than a transport convention. The TIR is managed and developed by International Road Transport Union (IRU) which is the world road transport organisation. The TIR procedure facilitates movement of goods under customs control across international borders without the payment of the duties and taxes. Justification: Statement 1: The TIR system secures customs duties and taxes and provides a robust guarantee mechanism, thereby reducing trade transaction costs, and facilitating higher growth of intra-regional and inter-regional trade. It does not eliminate those taxes and duties, which are otherwise dealt by trade agreements and WTO rules. Statement 2: On the eastern front, it will help India integrate with Myanmar and Thailand, as well as Bangladesh, Bhutan and Nepal. On the western front, it will enable India to move cargo along the International North-South Transport Corridor via Chabahar port in Iran, to access landlocked Afghanistan and the energy-rich Eurasian region. Q Source: http://www.livemint.com/Politics/dqvNmMJGbqQAC8pYZPAYdI/India-ratifies-TIR-Convention-to-acce ss-transnational-multi.html 18 The project "Financing of Energy Efficiency at MSMEs" in India is an initiative of A. World Bank and UN Energy Council B. Global Environmental Facility (GEF) and UNDP C. Bureau of Energy Efficiency (BEE), GEF and World Bank D. Green Climate Fund (GCF) and World Economic Forum (WEF) Your Answer : Correct Answer : C Answer Justification : Learning: The objective of the India-MSME Energy Efficiency Project is to improve efficiency and reduce GHG emissions through commercial investments in energy efficiency goods and services in target Small and Medium Enterprise clusters. The project aspires to address the current gap in understanding between energy auditors and bank loan officers. Q Source: http://pib.nic.in/newsite/PrintRelease.aspx?relid=155311 19 There are a number of ways for recording votes in the Parliament. The method of recording of votes by members on ‘Aye’ and ‘No’ slips is generally resorted to in which of the following eventualities? 1. When voting on any motion raised by a private member of Lok Sabha 2. At the commencement of the new Lok Sabha before seats/division numbers have been allotted to members Which of the above is/are correct? A. 1 only B. 2 only C. Both 1 and 2 D. None Your Answer : Correct Answer : B Answer Justification : Learning: The various methods adopted for voting in the Lok Sabha are: A) Voice Vote (aye i.e. yes or no) B) Division: There are three methods of holding a Division, i.e. By operating the Automatic Vote Recording Equipment; by distributing ` Ayes' and ` Noes' slips in the House; and by members going into the Lobbies. The Speaker directs the members for "Ayes" to go to the right Lobby and those for "Noes" to the left Lobby where their votes are recorded. C) Secret Ballot: Covered in tests earlier D) Recording of votes by distribution of slips: which is done in the case of statement 2 and when the automatic voice recording equipment has failed Q Source: Based on topics from Test 2 (MHRD schemes) 20 Many researchers and historians have opined that the Paika Rebellion of 1817 was India’s first organized armed rebellion against British Raj. Who were the Paiks? A. Traditional landed militia of Odisha B. Sepoys recruited by the Uttar Pradesh regiment of the British army C. Absentee landlords of Malabar D. Landless tribal peasants of Central India Your Answer : A Correct Answer : A Answer Justification : Learning: Paika rebellion predates the 1857 Revolt i.e. first war of independence. Paiks were the traditional landed militia of Odisha, used to perform policing functions. They owned rent-free land that was given to them for their military service to Kingdom of Khurda. However, by the policies of British East India Company, it was forcefully usurped rendering them landless. They were also subjected to repressive land revenue policies and humiliation by the British. At that critical juncture, Bakshi Jagabandhu Bidyadhar, the military chief of the King of Khurda, led army of Paikas forcing the East India Company to retreat. You can read a detailed description at the Q Source. Q Source: http://pib.nic.in/newsite/PrintRelease.aspx?relid=168762 21 India is the third largest Salt producing Country in the World after China and USA with Global annual production being about 230 million tonnes. Apart from sea brine, salt is derived in India from which of the following sources? 1. Sub-soil brine 2. Rock salt deposits 3. Lake brine 4. Groundwater in limestone caves Select the correct answer using the codes below. A. a) 1, 2 and 3 only B. b) 2 and 4 only C. c) 1 and 3 only D. d) 1, 2, 3 and 4 Your Answer : Correct Answer : A Answer Justification : Background: When India attained Independence in 1947, salt was being imported from the United Kingdom & Adens to meet its domestic requirement. But today it has not only achieved self-sufficiency in production of salt to meet its domestic requirement but also in a position of exporting surplus salt to foreign countries. Justification: Statement 1: Inland Salt Works in Rajasthan using lake brine and sub-soil brine viz. Sambhar Lake, Nawa, Rajas, Kuchhaman, Sujangarh and Phalodi. Salt works in Rann of Kutch using sub-soil brine viz: Kharaghoda, Dhrangadhra; Santalpur. Statement 2: Rock Salt Deposits at Mandi in the State of Himachal Pradesh Learning: Marine Salt works can be found along the coast of Gujarat, Tamil Nadu, Andhra Pradesh, Maharashtra, Orissa and West Bengal. Q Source: http://pib.nic.in/newsite/PrintRelease.aspx?relid=169773 22 Consider the following statements. National Academic Depository (NAD) 1. is an E-network of libraries in India 2. Facilitates recognition of degrees awarded by one institute by the other 3. Holds the repository of all academic talent in India for better resource utilization Select the correct answer using the codes below. A. 1 only B. 2 and 3 only C. 2 only D. 3 only

Correct Answer : C Answer Justification : Justification: NAD is a National System set-up by MHRD and UGC to facilitate Academic Institutions to Digitally, Securely and Quickly issue Online Academic Awards to the Students directly in their online NAD Account. The student can access certificate at any time and authorise employers, banks to view and verify the certificates. No more need to carry originals, submit attested photocopies, wait for verification of documents all the time. NAD aspires to make the vision of Digital Academic Certificates for every Indian a reality. NAD promises to do away with difficulties / inefficiencies of collecting, maintaining, and presenting physical paper certificates. Q Source: AKT/NT: (Release ID :167264) 23 Under the constitution of India, Sports is a A. Union Subject B. State subject C. Concurrent List subject D. Residuary list subject

Correct Answer : B Answer Justification : Learning: Sports is a State subject and, therefore, it is primarily the responsibility of the State Government to promote and develop Sports in the country. The Central Government complements/supplements the efforts of the State Governments. Under the revamped Khelo India proposal, there is an exclusive component called “Promotion of Rural, Indigenous and Tribal Games” for promotion and development of traditional sports. Q Source: http://pib.nic.in/newsite/PrintRelease.aspx?relid=169353 24 With reference to key initiatives taken by the Department of Expenditure, Ministry of Finance, in the last few years, consider the following. 1. All schemes and projects have now an Outcome Framework. 2. Each Scheme will now have a start and a sun-set date co-terminus with the Finance Commission Cycle. 3. Number of both central Sector and Centrally Sponsored Schemes were increased for improved coordination between implementing agencies. Select the correct answer using the codes below. A. 1 only B. 1 and 2 only C. 2 and 3 only D. 3 only

Correct Answer : B Answer Justification : Justification: Statement 1 and 2: An outcome framework focuses on the impact of the scheme, rather than only the outlay (finances) of the scheme. Schemes and projects have now an Outcome Framework, which has been formulated through consultation with the implementing Ministries/ Departments, NITI Aayog and the Department of Expenditure. A consolidated Outcome Budget 2017-18 was presented in the Parliament as a part of the Budget Documents. Third party Evaluation has also been formally built into the appraisal and approval framework to ensure periodic feedback, a more concerted focus on outcomes and enhanced growth performance Statement 3: Both Central Sector and Centrally Sponsored Schemes were rationalized (reduced and consolidated significantly in number) in consultation with the line Ministries. Q Source: http://pib.nic.in/newsite/PrintRelease.aspx?relid=161801 25 Recently seen in news, QueSST is A. A Quiet Supersonic transport aircraft design by NASA B. An interstellar mission of Space X C. A new technology for improving air flow in scuba diver suits D. A method for breaking non-biodegradable enzymes

Correct Answer : A Answer Justification : Learning: NASA has reached a major milestone in developing a quieter supersonic passenger jet that is capable of safely travelling over land. The project falls under the ambit of NASA’s Commercial Supersonic Technology Project. Quiet Supersonic Transport (QueSST) aircraft design is the initial design stage of NASA’s planned Low Boom Flight Demonstration (LBFD) experimental airplane. The LBFD is also known as X-plane. With this development, NASA is a step closer to actually building an X-plane. Q Source: https://www.nasa.gov/feature/the-quesst-for-quiet/ 26 After consolidating their position in India, the Sultans introduced reforms in the land revenue administration. In this context, inam lands were assigned to A. officials against the payment for their services B. religious leaders or religious institutions C. courtyards for mosques D. expand the market in the hinterland of the empire

Correct Answer : B Answer Justification : Learning: The lands were classified into three categories: iqta land – lands assigned to officials as iqtas instead of payment for their services. khalisa land – land under the direct control of the Sultan and the revenues collected were spent for the maintenance of royal court and royal household. inam land – land assigned or granted to religious leaders or religious institutions. However, despite the land reforms, the peasantry paid one third of their produce as land revenue, and sometimes even one half of the produce. They also paid other taxes and always led a hand-tomouth living. Q Source: 11th TN Textbook 27 Consider the following statements. Tasawwuf 1. stressed the elements of love and devotion as effective means of the realisation of God 2. considered rituals more important than self-discipline in reaching the highest goal Which of the above is/are correct? A. 1 only B. 2 only C. Both 1 and 2 D. None

Correct Answer : A Answer Justification : Justification: Statement 1 and 2: Tasawwuf is another name for Sufism, as it is known in the Muslim world. In Sufism, self-discipline was considered an essential condition to gain knowledge of God by sense of perception. While orthodox Muslims emphasise external conduct and rituals, the Sufis lay stress on inner purity. According to them one must have the guidance of a pir or guru, without which spiritual development is impossible. Q Source: 11th TN Textbook 28 As per the recent amendments to the Legal Metrology (Packaged Commodities) Rules, 2011; starting from the year 2018 1. It would become mandatory for medical devices to print/label MRP on the packages. 2. Every medical package shall bear the contact details of the person who can be contacted in case of consumer complaints. Which of the above is/are correct? A. 1 only B. 2 only C. Both 1 and 2 D. None

Correct Answer : C Answer Justification : Justification: These amendments were made by the Ministry of Consumer Affairs, Food and Public Distribution making these rules applicable to medical devices, on one hand, and to e-commerce companies, on the other. Statement 1: The e-commerce companies are required to ensure that the mandatory declarations are displayed on the digital and electronic network used for e-commerce transactions viz. Name and address of the manufacturer/ packer/ Importer, Name of the Commodity, Net quantity, Retail sale price in the form MRP (inclusive of all taxes) and consumer care details. Statement 2: It shall bear the name, address, telephone number, e-mail address of the person who can be or the office which can be contacted in case of consumer complaints. Learning: The Department of Consumer Affairs had also launched a portal “Grievance against Misleading Advertisement (GAMA)” in 2015 where a consumer can lodge a complaint against a misleading advertisement. Q Source: http://consumeraffairs.nic.in/forms/contentpage.aspx?lid=639 29 The India Newborn Action Plan (INAP) was launched with the aim of 1. Ending preventable newborn deaths and stillbirths by 2030 2. Reducing fertility rate to zero by 2030 3. Increasing Aanganwadi enrolment of children of 0-6 years of age to cent per cent Select the correct answer using the codes below. A. 1 and 2 only B. 1 only C. 2 only D. 2 and 3 only

Correct Answer : B Answer Justification : Justification: The salient features are: INAP has set the goals for neonatal mortality and stillbirths. The goal is to attain Single Digit Neonatal Mortality and Stillbirth Rates by 2030. INAP is to be implemented within the existing Reproductive, Maternal, Newborn, Child and Adolescent health (RMNCH+A) framework of the National Health Mission (NHM). Q Source: http://www.business-standard.com/article/news-cm/as-per-the-national-family-health-survey-nfhs-4-2 015-16-35-7-per-cent-children-below-five-years-are-underweight-117081101085_1.html 30 Consider the following statements. 1. Kanishka sent to Central Asia and China for the propagation of Buddhism. 2. Kanishka patronised not only Buddhist scholars like Vasumitra, but also patronised the famous physician of ancient India, Charaka. Which of the above is/are correct? A. 1 only B. 2 only C. Both 1 and 2 D. None

Correct Answer : C Answer Justification : Justification: Statement 1: This is because Mahayana Buddhism came in vogue during Kanishka’s rule. It was established in the Fourth Buddhist Council convened by him where matters relating to Buddhist theology and doctrine were discussed. In Mahayana Buddhism, the Buddha came to be worshipped with flowers, garments, perfumes and lamps. Statement 2: Asvagosha and were other important scholars patronized by him. Q Source: 11th TN Textbook 31 The RAISE Act, recently seen in news, in the context of United States of America, has a bearing on A. India-USA defence relations B. Indian Alliance with NAFTA C. Cross border smuggling of arms and ammunition in India D. Emigration of workers and professionals from India to the USA

Correct Answer : D Answer Justification : Learning: USA President Trump has announced his support for the Reforming American Immigration for Strong Employment (RAISE) Act that would cut in half the number legal immigrants allowed into the US while moving to a “-based” system favouring English-speaking skilled workers for residency cards. The RAISE Act would scrap the current lottery system to get into the US and instead institute a points-based system for earning a green card. Factors that would be taken into account include English language skills, education, highpaying job offers and age. The RAISE Act ends chain migration, and replaces the low-skilled system with a new pointsbased system for receiving a Green Card. 32 Consider the following statements with reference to Jaina sects. 1. The Svetambara tradition of Jainism trace their through Sthulabhadra. 2. As per the Digambara sect of Jainism, Bhadrabahu was the last Shruta Kevalin. Which of the above is/are correct? A. 1 only B. 2 only C. Both 1 and 2 D. None

Correct Answer : C Answer Justification : Justification & Learning: Statement 1: By the end of the fourth century B.C., there was a serious famine in the valley. Many Jain monks led by Bhadrabagu and Chandragupta Maurya came to Sravana Belgola in Karnataka. Those who stayed back in north India were led by a monk named Sthulabahu who changed the code of conduct for the monks. This led to the division of Jainism into two sects Svetambaras (whiteclad) and Digambaras (Sky-clad or Naked). The first Jain Council was convened at by Sthulabahuin the beginning of the 3rd century B.C. Statement 2: According to the Digambara sect of Jainism, there were five Shruta Kevalins in Jainism - Govarddhana Mahamuni, Vishnu, Nandimitra, Aparajita and Bhadrabahu. Shrutakevalin is a term used in Jainism for those ascetics who have complete knowledge of Jain Agama (texts). Q Source: Additional Research: 11th TN Textbook 33 Ahar-Pyne system, recently seen in news, is a A. Traditional floodwater harvesting system in South Bihar B. Plant extract used in addressing anemia among tribals C. System of rock mining in earthquake prone regions D. Free national meal scheme launched by an NGO

Correct Answer : A Answer Justification : Justification: Ahar-pyne system is an indigenous irrigation technology. This system has evolved from an understanding of the particular agro-climatic conditions of the South Bihar plains of India. Ahars are reservoirs and consist of a major embankment across the line of the drainage with two side embankments running backwards up to the line of the drainage gradually losing their heights because of the gradient of the surface. Thus, an ahar resembles a rectangular catchment basin with only three embankments, and the fourth side left open for the drainage water to enter the catchment basin following the natural gradient of the country. Learning: A model of water conservation adopted successfully by the authorities in Nalanda district of south central Bihar, has been selected for the national award for excellence in the Mahatma Gandhi national rural employment guarantee programme (MGNREGP), by the ministry of rural development. The award for excellence will be conferred on ‘Project Jal Sanchay’, the water conservation model. ‘Project jal sanchay’ was launched under MGRNREGP, to offer farmers a wide spectrum of solutions to water crisis. Q Source: As mentioned above 34 ‘We Wear Culture’, recently seen in news, is a A. Global digital platform fashion project launched by Google B. Commemoration of the Silk Road organized by Ministry of Culture, India C. Awareness campaign against the use of Silk clothes that involve killing of live cocoons D. A dress code for civil servants released by the Office of President of India

Correct Answer : A Answer Justification : Learning: Working with hundreds of renowned cultural institutions from around the world, including India, Google’s project ‘We Wear Culture’ lets people explore the history of clothes. It covers dates as early as 3,000 years ago from the ancient Silk Road, to the courtly fashion of Versailles, to the elegance of the Indian Sarees. The world fashion exhibit also showcases designs from north-eastern India including the weaves of tribes such as the Nagas, Meitis and the traditional attire from Meghalaya called ‘Dhara’ or ‘Nara’ worn by the Khasi women. Q Source: http://www.business-standard.com/article/current-affairs/patola-temple-and-ikat-sarees-part-of-goog le-s-we-wear-culture-project-117060800664_1.html 35 Consider the following with reference to the Fa Hien’s account of India. 1. During his visit, veneration was not practiced in India. 2. He frequently mentions the name of Chandragupta II in his works. 3. As per him, Buddhism was established mainly in the Gangetic valley. 4. He was looking for better copies of Buddhist books than were available in China at that time. Select the correct answer using the codes below. A. 2 and 3 only B. 4 only C. 3 only D. 1, 2, 3 and 4

Correct Answer : B Answer Justification : Justification: Statement 4: Between 399 and 414 CE, the Chinese monk Fa Hien undertook a trip via Central Asia to India seeking better copies of Buddhist books than were currently available in China. Statement 2: As far as the Gupta empire is concerned, although his account is valuable in many respects, he did not mention the name of Chandragupta II. He was not interested in political affairs. His interest was primarily religion. Statement 3: According to him, Buddhism was in a flourishing condition in the north-western India but in the Gangetic valley it was in a state of neglect. He refers to the Gangetic valley as the ‘land of Brahmanism’. Statement 1: He indicates clearly the importance of the seven precious substances for Buddhist worship, the widespread practice of stupa veneration, and his aquaintance with several of the jataka tales about the previous lives of the Buddha Sakyamuni. Q Source: TN 11th Standard Textbook 36 The National Tiger Conservation Authority (NTCA) is set up under the Chairmanship of the A. Prime Minister of India B. Minister for Environment, Forests and Climate Change (MoEFCC) C. Director, Wildlife Institute of India (WII) D. Secretary to the Government of India, MoEFCC

Correct Answer : B Answer Justification : Learning: NTCA is a statutory body under the MoEFCC constituted under enabling provisions of the Wildlife (Protection) Act, 1972, as amended in 2006, for strengthening tiger conservation, as per powers and functions assigned to it under the said Act. The Authority, interalia, would lay down normative standards, guidelines for tiger conservation in the Tiger Reserves, apart from National Parks and Sanctuaries. It would provide information on protection measures including future conservation plan, tiger estimation, disease surveillance, mortality survey, patrolling, report on untoward happenings and such other management aspects as it may deem fit, including future plan for conservation. Q Source: Frequently seen in news 37 With reference to Chalcolithic cultures in Indian subcontinent, consider the following statements. 1. Ochre-coloured pottery sites have been usually found in the Gangetic doab of India. 2. Anthropomorphic figures of worship have been found in the copper hoard culture. Which of the above is/are correct? A. 1 only B. 2 only C. Both 1 and 2 D. None

Correct Answer : C Answer Justification : Concept: The Harappan culture is generally supposed to have been followed by non-urban Chalcolithic culture characterized by the use of copper and stone. The differences between these cultures were not fundamental but were primarily confined to pottery. Justification: Copper Hoards describe find-complexes which occur in the northern part of India. These occur mostly in hoards large and small and are believed to date to the later 2nd millennium BCE, although very few derive from controlled and dateable excavation contexts. A fragment of an anthropomorph came to light in controlled excavations at Lothal and a second one at Saipai Lichchwai, Etawah district. The doab hoards are associated with the so-called Ochre Coloured Pottery (OCP) which appears to be closely associated with the Late Harappan (or Posturban) phase. Q Source: Additional Research: 11th TN Textbook 38 The main functions of Small Farmers’ Agri-Business Consortium (SFAC) established by the Government include 1. Helping formation and growth of Farmer Producer Organizations (FPOs) 2. Implementation of National Agriculture Market (e-NAM) Electronic Trading platform 3. Improving availability of working capital and development of business activities of Farmer Producer Companies (FPCs) Select the correct answer using the codes below. A. 2 only B. 1 and 3 only C. 3 only D. 1, 2 and 3

Correct Answer : D Answer Justification : Justification: It was established as a Society in 1994 to facilitate agri-business ventures by catalysing private investment through Venture Capital Assistance (VCA) Scheme in close association with financial institutions. The role of State SFACs is to aggressively promote agribusiness project development in their respective States. Q Source: SS (Release ID :169745) 39 The Clean Energy Ministerial (CEM) is 1. A high-level global forum to share lessons learnt and best practices for the transition to a global clean energy economy 2. An initiative of the World Trade Organization (WTO) 3. Financing Green Carbon Credits (GCC) allotted to Least Developed Countries (LDCs) 4. The main implementing agency for the programs of the United Nations Environment Programme (UNEP) Select the correct answer using the codes below. A. 1, 2 and 3 only B. 2 and 4 only C. 1 only D. 2, 3 and 4 only

Correct Answer : C Answer Justification : Justification: Statement 2: The CEM is the initiative of the USA. Presently, 23 countries are members of CEM. There are 13 initiatives under CEM covering energy efficiency, clean energy supply and cross cutting areas. Statement 3: It does not finance carbon credits. Concept: A carbon credit is a generic term for any tradable certificate or permit representing the right to emit one tonne of carbon dioxide or the mass of another greenhouse gas with a carbon dioxide equivalent (tCO2e) equivalent to one tonne of carbon dioxide. Rest: Since its launch in 2010, it has launched a number of initiatives that are based on areas of common interest among participating Governments and other stakeholders. Q Source: http://www.thehindubusinessline.com/specials/clean-tech/irena-adnan-z-amin-solar-energy-wind-ene rgy-china/article9731412.ece 40 In-STEP program of Ministry of Human Resource Development (MHRD) works towards A. Capacity Building Program of Teacher Educators B. Open access for University education in India C. Documenting traditional literary heritage of India D. Improving tribal literacy

Correct Answer : A Answer Justification : Learning: Under In-STEP programme (India Support to Teacher Education Program) several teacher educators from the Eastern and North-Eastern part of the country have undergone fellowships to understand issues related to teacher education. In addition, the TESS-India is another programme for Teacher Educators with collaboration between Department for International Development (DFID) and Ministry of HRD with UK’s Open University as the implementing agency. Learning: Digital Gender Atlas for Advancing Girls’ Education in India is a tool that helps identify low performing geographic pockets for girls, particularly from marginalised groups. Q Source: Based on topics from Test 2 (MHRD schemes) 41 National Water Development Agency (NWDA) carries out which of these functions? 1. Carry out detailed surveys and investigations of possible reservoir sites and interconnecting links for peninsular rivers 2. Monitor, regulate and control water flows between inter-state river basin embankments 3. Assessing groundwater potential in the country and making adequate arrangements for exploration of groundwater Select the correct answer using the codes below. A. 1 only B. 2 only C. 1 and 3 only D. 1, 2 and 3 Your Answer : Correct Answer : A Answer Justification : Justification: It was set up in July, 1982 as Autonomous Society under the Societies Registration Act, 1860, to carry out the water balance and other studies on a scientific and realistic basis for optimum utilisation of Water Resources. Functions include: To prepare feasibility report of the various components of the scheme relating to Peninsular Rivers development and Himalayan Rivers development. To prepare detailed project report of river link proposals under National Perspective Plan for Water Resources Development after concurrence of the concerned States. Q Source: Samir/JD/jk (Release ID :169810) 42 Anthanars and thuthars in the Sangam monarchy were A. Priests and envoys B. Spies and military commanders C. Custom officials and seaport traders D. Ministers and army officials Your Answer : Correct Answer : A Answer Justification : Learning: The imperial court or avai was attended by a number of chiefs and officials. The king was assisted by a large body of officials who were divided into five councils. They were ministers (amaichar), priests (anthanar), military commanders (senapathi), envoys (thuthar) and spies (orrar). Each ruler had a regular army and their respective Kodimaram (tutelary tree). Land revenue was the chief source of state’s income while custom duty was also imposed on foreign trade. The Pattinappalai refers to the custom officials employed in the seaport of Puhar. Q Source: 11th TN History Textbook 43 Consider the following statements. Assertion (A): Pallava rulers issued their charters only in Prakrit. Reason (R): Pallava rulers were against the dominance of Brahmanism and banned the use of for all official works. In the context of the above, which of these is correct? A. A is correct, and R is an appropriate explanation of A. B. A is correct, but R is not an appropriate explanation of A. C. A is correct, but R is incorrect. D. Both A and R are incorrect. Your Answer : Correct Answer : D Answer Justification : Justification: The early Pallava rulers from 250 A.D. to 350 A.D. issued their charters in Prakrit. Important among them were Sivaskandavarman and Vijayaskandavarman. The second line of Pallava rulers who ruled between 350 A.D. and 550 A.D. issued their charters in Sanskrit. So, R is wrong. Q Source: 11th TN Textbook 44 NASA’s ROSA is expected to 1. Detect asteroid clashes in outer space 2. Power satellites and spacecraft in the future 3. Feed the International Space Station (ISS) with biological supplies Select the correct answer using the codes below. A. 1 and 2 only B. 3 only C. 2 only D. None of the above Your Answer : Correct Answer : C Answer Justification : Concept: NASA is testing the effectiveness of a flexible solar array, the Roll-Out Solar Array, or ROSA, on space station for the very first time. It was tested on earth earlier. It rolls out like a tape measure. ROSA is smaller and lighter than the traditional panels and has a centre wing build of a flexible material containing photovoltaic cells to convert light into electricity. Application: ROSA is expected to provide power for a variety of future spacecraft and satellites. It will be an asset for satellite radio and television, weather forecasting, GPS and other services used on the Earth. The NASA engineers remotely attached the ROSA to the International Space Station’s robotic Canadarm2. ROSA will remain attached to the Canadarm2 over seven days to test its effectiveness. Scientists will test the effectiveness of ROSA in the microgravity and extreme temperatures of space. Q Source: https://www.nasa.gov/feature/roll-out-solar-array-technology-benefits-for-nasa-commercial-sector/ 45 The ‘Inclusive India Initiative’ of National Trust is specifically catering to A. persons with intellectual and developmental disabilities B. women from underprivileged backgrounds C. children living in tribal areas who have poor access to health and education D. None of the above Your Answer : A Correct Answer : A Answer Justification : Learning: The Inclusive India initiative is an attempt to facilitate the realisation of equal opportunities, protection of rights and full participation of individuals with intellectual and developmental disabilities into the fabric of society. The three core focus areas of Inclusive India Initiative are Inclusive Education, Inclusive Employment and Inclusive Community Life. Q Source: http://pib.nic.in/newsite/PrintRelease.aspx?relid=164434 46 He was a staunch follower of Brahmanism. Buddhist sources often refer him as a persecutor of Buddhism. He is? A. Pushyamitra B. Menander C. Somapura D. Dharmaraksita Your Answer : Correct Answer : A Answer Justification : Learning: He belonged to the Sunga dynasty. The rule of the Sungas was important because they defended the Gangetic valley from foreign invasions. In the cultural sphere, the Sungas revived Brahmanism and horse sacrifice. They also promoted the growth of Vaishnavism and the Sanskrit language. “In short, the Sunga rule was a brilliant anticipation of the golden age of the Guptas” - This statement can be asked as a question by UPSC in Mains. Clarification: While Buddhist sources refer him as a persecutor of Buddhism. But there is enough evidence to show that Pushyamitra patronised . During his reign, the Buddhist monuments at Bharhut and were renovated and further improved. Q Source: 11th TN History Textbook 47 The taking of Census was regular during the Mauryan period. Consider the following with reference to it. 1. Village officials were to record the caste and occupation of people. 2. Livestock population in each house was tracked. 3. Municipal officials tracked the movement of population both foreign and indigenous. 4. The data collected from official census were cross checked by the spies. Select the correct answer using the codes below. A. 1 and 2 only B. 2, 3 and 4 only C. 3 and 4 only D. 1, 2, 3 and 4 Your Answer : Correct Answer : D Answer Justification : Justification: This is a lateral explanation, since the details of census have already been covered in the question. We will touch upon other important aspects of Mauryan administration. Other aspects: Kautilya mentions the existence of both civil and criminal courts. The chief justice of the Supreme Court at the capital was called Dharmathikarin. There were also subordinate courts at the provincial capitals and districts under Amatyas. Different kinds of punishment such as fines, imprisonment, mutilation and death were given to the offenders. Torture was employed to extract truth. Police stations were found in all principal centres. Both Kautilya and Asokan Edicts mention about jails and jail officials. Remission of sentences is also mentioned in Asoka’s inscriptions. Q Source: 11th TN History Textbook 48 Within the Earth's ocean and atmosphere, Rossby waves form as a result of A. Rotation of the planet B. Churning of oceans C. Tectonic movement beneath ocean plates D. Collision of clouds Your Answer : A Correct Answer : A Answer Justification : Learning: Waves in the ocean come in many different shapes and sizes. Slow-moving oceanic Rossby waves are fundamentally different from ocean surface waves. Unlike waves that break along the shore, Rossby waves are huge, undulating movements of the ocean that stretch horizontally across the planet for hundreds of kilometers in a westward direction. Also known as planetary waves, they naturally occur in rotating fluids. They are so large and massive that they can change Earth's climate conditions. Along with rising sea levels, King Tides, and the effects of El Niño, oceanic Rossby waves contribute to high tides and coastal flooding in some regions of the world. Q Source: Important Concepts in Geography 49 In the Gupta empire, Paramabhattaraka was a A. Minister for foreign affairs B. Title adopted by Gupta Kings C. Council of Peace and conflict D. In charge of granaries Your Answer : Correct Answer : B Answer Justification : Learning: According inscriptions, the Gupta kings assumed titles like Paramabhattaraka, Maharajadhiraja, Parameswara, Samrat and Chakravartin. This is a lateral explanation, and we will go into other details of Gupta administration. Other details: The king was assisted in his administration by a council consisting of a chief minister, a Senapati or commander in-chief of the army and other important officials. A high official called Sandivigraha was mentioned in the Gupta inscriptions, most probably minister for foreign affairs. The king maintained a close contact with the provincial administration through a class of officials called Kumaramatyas and Ayuktas. Provinces in the Gupta Empire were known as Bhuktis and provincial governors as Uparikas. Q Source: 11th TN History Textbook 50 With reference to Convention on International Civil Aviation, also known as the Chicago Convention, consider the following statements. 1. It establishes rules for the safety of airlines. 2. The convention exempts air fuels in transit from double taxation. 3. It provides that “Every state has complete and exclusive sovereignty over airspace above its territory”. 4. International Civil Aviation Organization (ICAO) was established by this convention. Select the correct answer using the codes below. A. 1 and 2 only B. 3 and 4 only C. 2 and 3 only D. 1, 2, 3 and 4 only Your Answer : Correct Answer : D Answer Justification : Justification: Statement 1: The Convention establishes rules of airspace, aircraft registration and safety, and details the rights of the signatories in relation to air travel. Statement 2: Air transits would tend to impose tax twice on the aviation fuel if not checked for double taxation. This is similar to how MNCs would be double taxed if a DTAA hasn’t been signed. Statement 3: Every other State must refrain from resorting to the use of weapons against civil aircraft in flight. Statement 4: ICAO is a UN specialized agency, established by States in 1944 to manage the administration and governance of the Convention on International Civil Aviation (Chicago Convention). Q Source: ICAO is frequently in news 51 With reference to Ancient India, according to Manu 1. Father could not give his wealth to daughter as a gift. 2. Mother’s wealth would pass to the daughters. 3. Women could receive wealth in Adhyavahanika. Select the correct answer using the codes below. A. 1 only B. 2 and 3 only C. 3 only D. 1, 2 and 3 Your Answer : D Correct Answer : B Answer Justification : Justification: Statement 1: Manu did not project that daughters are not the natural heirs of their fathers, but the father was at liberty to give wealth to daughter as gift. Manu also refers to the putrika or daughter’s son functioning as an heir for a sonless father. Statement 2: The mother’s wealth would pass to the daughters, according to the MS. The different occasions in the MS when gifts can be given to the daughter, over which she has right were: Adhyagni – before the nuptial fire; Adhyavahanika – in the bridal procession; Pritidatta – out of love; Bhratrimatripitripraptam- received from brother, mother, father Q Source: http://vle.du.ac.in/mod/book/view.php?id=5353&chapterid=1347 52 Naganandam written by Harsha deals with A. Story of a B. Accounts of social justice in the Harsha empire C. Love story of an emperor D. The first record of a snake deity Your Answer : A Correct Answer : A Answer Justification : Learning: Harsha wrote three plays--the Priyadarshika, the Ratnavali and the Naganandam. The first two are ‘sister plays’, dealing with harem intrigue, and the third tells the story of Jimutavahana, a bodhisattva. Jimutavahana (c. 12th century) was an Indian Sanskrit scholar and writer of legal and religious treatises of early medieval period. He was the earliest writer on smriti (law) from Bengal whose texts are extant. Q Source: 11th TN History Textbook 53 Gaj Yatra Campaign is being organized by A. Wildlife Trust of India (WTI) B. International Union for Conservation of Nature (IUCN) C. United Nations Environment Programme (UNEP) D. World Wildlife Fund (WWF) Your Answer : Correct Answer : A Answer Justification : Learning: It is a 15 months nationwide campaign led by the WTI to protect elephants on the occasion of World Elephant Day. During the period of the campaign, artists and craftsmen will create life-size works on the theme of elephants in places along the route of the roadshow, covering 12 states that have wild elephants, using local art and craft. Specially fabricated vehicles will be deployed to display these on pre-determined routes with campaigners. Q Source: http://pib.nic.in/newsite/PrintRelease.aspx?relid=169934 54 What do you understand by ‘protectionism’, a term often seen in newspapers? A. Imposing trade restrictions to protect domestic industry B. Entering into alliances with foreign militaries to safeguard one’s borders C. Mandating military conscription for better defence capabilities D. Protecting one’s citizen from influence of foreign cultures Your Answer : Correct Answer : A Answer Justification : Learning: In economics, protectionism is the economic policy of restraining trade between states (countries) through methods such as tariffs on imported goods, restrictive quotas, and a variety of other government regulations. Protectionist policies protect the producers, businesses, and workers of the import-competing sector in a country from foreign competitors. However, they hurt consumers in general, and the producers and workers in export sectors, both in the country implementing protectionist policies, and in the countries protected against. Q Source: 55 The Monetary Policy Committee (MPC) of the Reserve Bank of India (RBI) has reduced short-term lending rate, or repo rate, by 25 basis points to 6%. What may be the impact of this move on the banking system? 1. Draining of foreign exchange reserves 2. Pickup in short-term borrowing due to lowering of interest rates 3. Sudden increase in international trade and commerce of India Select the correct answer using the codes below. A. 1 and 2 only B. 3 only C. 2 and 3 only D. 2 only Your Answer : C Correct Answer : D Answer Justification : Justification: Statement 1: Domestic borrowing and lending do not directly affect the forex reserves. Statement 2: Repo rate represents the short-term borrowing cost of banks. If it is reduced, banks pass on the lowered borrowing cost to consumers. Statement 3: There may be an increase in trade and commerce as lowered interest rates lead to higher economic activity and greater exports. However, this will not be a sudden impact, it will take time. Q Source: As mentioned above 56 World’s first Indian Partition Museum is situated in A. Amritsar B. New Delhi C. Kolkata D. Lahore Your Answer : A Correct Answer : A Answer Justification : Learning: The Museum is part of the newly inaugurated Heritage Street at Amritsar, which starts at the Golden Temple and ends at the Town Hall. The Partition Museum Project (TPMP) was initiated by The Arts and Cultural Heritage Trust (TAACHT) to work towards the establishment of a world class, physical museum, dedicated to the memory of the Partition of the sub-continent in 1947 — its victims, its survivors and its lasting legacy. Q Source: http://www.partitionmuseum.org/about-us/ 57 Hyperspectral imaging can be useful in 1. Finding oil fields 2. Monitoring crop health 3. Diagnosis of retinopathy 4. Removing defects and foreign material in processed food Select the correct answer using the codes below. A. 1 and 2 only B. 2, 3 and 4 only C. 3 and 4 only D. 1, 2, 3 and 4 Your Answer : Correct Answer : D Answer Justification : Concept: Whereas the human eye sees colour of visible light in mostly three bands (red, green, and blue), spectral imaging divides the spectrum into many more bands. This technique of dividing images into bands can be extended beyond the visible. In hyperspectral imaging, the recorded spectra have fine wavelength resolution and cover a wide range of wavelengths. This can cover very fine imaging. Justification: Statement 1: Known as spectral signatures, these 'fingerprints' enable identification of the materials that make up a scanned object. For example, a spectral signature for oil helps geologists find new oil fields. Statement 2: Although the cost of acquiring hyperspectral images is typically high, for specific crops and in specific climates, hyperspectral remote sensing use is increasing for monitoring the development and health of crops. This is because the finer colours that indicate crop health can be mapped by hyperspectral imaging. Statement 3: Researchers in Montraal are working to test the use of hyperspectral photography in the diagnosis of retinopathy and macular edema before damage to the eye occurs. Statement 4: In the food processing industry, hyperspectral imaging, combined with intelligent software, enables digital sorters (also called optical sorters) to identify and remove defects and foreign material (FM) that are invisible to traditional camera and laser sorters. Q Source: ISRO) is planning to launch a full-fledged niche Earth observation (EO) satellite — called the Hyperspectral Imaging Satellite (HySIS) 58 Tropic of Cancer cuts which of these Asian rivers? A. Indus B. Xi Jiang C. Chang Jiang D. Godavari Your Answer : Correct Answer : B Answer Justification : Justification: Q Source: Map based questions: South-east Asia 59 The 2030 Water Resources Group is 1. A inter-governmental forum founded by United Nations Environment Programme (UNEP) 2. Hosted by the International Finance Corporation (IFC) 3. Focusing on water resources reform in water stressed countries Select the correct answer using the codes below. A. 1 only B. 2 and 3 only C. 3 only D. 1, 2 and 3 Your Answer : Correct Answer : B Answer Justification : Justification: Statement 1 and 3: It is a unique public-private-civil society collaboration. The 2030 WRG was launched in 2008 at the World Economic Forum and has been hosted by IFC since 2012. Statement 2: It facilitates open, trust-based dialogue processes to drive action on water resources reform in water stressed countries in developing economies. The ultimate aim of such reforms and actions is to close the gap between water demand and supply by the year 2030. Our global partners include companies like PepsiCo, Coca-Cola, development banks the World Bank, African Development Bank and INGOs and IGPs like UNDP, IUCN. Q Source: 2030WRG) has decided to help the Maharashtra Government to raise $270 million from the Green Climate Fund (GCF).... 60 Which of these organizations in India has recently overtaken the responsibility of overseeing all the data collection related to Crime, Suicides and Accidental Deaths in India? A. National Crime Records Bureau (NCRB) B. Central Bureau of Investigation (CBI) C. Research and Analysis Wing (RAW) D. Bureau of Police Research and Development (BPRD) Your Answer : A Correct Answer : D Answer Justification : Learning: Recently, the three-decade old NCRB has been merged with another central police organisation - the BPRD with an aim to improve efficiency of development works related to policing. Now BPR&D will oversee all the data collection related to Crime in India, Suicides and Accidental Deaths, Prison Statistics and Fingerprints which was earlier done by NCRB. Q Source: http://www.business-standard.com/article/pti-stories/govt-merges-ncrb-with-bprd-117080600317_1. html 61 Consider the following with reference to major international dates related to environment. 1. The International Day for Biological Diversity was initiated by the United Nations General Assembly (UNGA). 2. Host nation for World Environment Day (WED) 2017 was India. 3. Earth Hour is celebrated at noon local time to acknowledge the importance of the solar cycle. 4. World Wetlands Day marks the date of the adoption of the Ramsar Convention on Wetlands. Select the correct answer using the codes below. A. 1 and 2 only B. 2, 3 and 4 only C. 1 and 4 only D. 1, 2, 3 and 4 Your Answer : C Correct Answer : C Answer Justification : Justification: Statement 1: It is a United Nations–sanctioned international day for the promotion of biodiversity issues. It is currently held on May 22. Statement 2: The host nation is Canada. The theme for 2017 is 'Connecting People to Nature – in the city and on the land, from the poles to the equator'. Statement 3: Earth Hour is a worldwide movement organized by the World Wide Fund for Nature (WWF). It is celebrated at 8:30pm (local time). Statement 4: It’ll be covered in detail later. Q Source: Various environmental dates appearing in news 62 India has launched one of the world’s largest vaccination campaign against measles and congenital rubella syndrome (CRS). These diseases are 1. Highly contagious and caused by bacteria 2. Responsible for irreversible birth defects in children even causing death Which of the above is/are correct? A. 1 only B. 2 only C. Both 1 and 2 D. None Your Answer : D Correct Answer : B Answer Justification : Justification: These are vaccine preventable diseases. Statement 1: Measles is a highly contagious disease caused by a virus. Measles can occur in a person who has never had measles and has no immunity against it. Statement 2: Congenital rubella syndrome (CRS) is an illness resulting from rubella virus infection during pregnancy. When rubella infection occurs during early pregnancy, serious consequences—such as miscarriages, stillbirths, and a constellation of severe birth defects in infants can result. Learning: The campaign aims to vaccinate more than 35 million children in the age group of nine months to 15 years with MR (measles and rubella) vaccine. For the MR campaign to be effective, it is important that throughout its duration, and in routine immunization thereafter, no child is left behind. Q Source: http://indianexpress.com/article/india/centre-completes-first-phase-of-the-measles-rubella-vaccinatio n-campaign-4787046/ 63 Which of these geographical regions are known as sub-continents? 1. Indian subcontinent 2. Arabian Peninsula 3. Greenland 4. Alaskan peninsula Select the correct answer using the codes below. A. 1 and 2 only B. 1, 3 and 4 only C. 2 and 3 only D. 1, 2, 3 and 4 Your Answer : A Correct Answer : D Answer Justification : Justification: Certain parts of continents are recognized as subcontinents, especially the large peninsulas separated from the main continental landmass by geographical features. The most notable examples are the Indian subcontinent and the Arabian Peninsula. The southern cone of South America and Alaskan peninsula of North America are other examples. In many of these cases, the "subcontinents" concerned are on different tectonic plates from the rest of the continent, providing a geological justification for the terminology. Greenland, generally reckoned as the world's largest island on the north-eastern periphery of the North American Plate, is sometimes referred to as a subcontinent. This is a significant departure from the more conventional view of a subcontinent as comprising a very large peninsula on the fringe of a continent. Q Source: 6th NCERT Geography: General questions 64 ‘Bharat 22’, recently seen in news, is expected to A. Hasten Government’s disinvestment programme B. Improve conservation of 22 scheduled languages in the Constitution of India C. Increase maritime and land security of all coastal and border states of India D. Reduce red tapism in the delivery of welfare schemes Your Answer : Correct Answer : A Answer Justification : Learning: This is an exchange traded fund (ETF) launched by Union Finance Ministry. It will help to speed up disinvestment programme budgeted to raise a record Rs 72,500 crore in the FY 2018. Bharat 22 spans six sectors — basic materials, energy, finance, industrials, FMCG and utilities. The sector wise weightage in the Bharat 22 Index is basic materials (4.4%), energy (17.5%), finance (20.3%), FMCG (15.2%), industrials (22.6%), and utilities (20%). Q Source: http://economictimes.indiatimes.com/markets/stocks/news/reasons-why-bharat-22-etf-can-be-a- winwin- deal-for-you/articleshow/60029997.cms 65 AGRI-UDAAN Food and Agribusiness Accelerator 2.0 programme will 1. Finance Gross Capital Formation (GCF) at subsidized rates for large land holdings 2. Setup Mega Food Parks in all smart villages of India 3. Mentor related start-ups and help them connect with potential investors Select the correct answer using the codes below. A. 1 only B. 2 and 3 only C. 3 only D. 1, 2 and 3 Your Answer : Correct Answer : C Answer Justification : Justification: It intends to promote innovation and entrepreneurship in agriculture. It will mentor start-ups and help them connect with potential investors and thus convert innovative ideas from India’s rural youth into viable businesses. It is managed by ICAR-NAARM’s (Indian Council of Agricultural Research-National Academy of Agricultural Research Management). Under the programme, start-ups will get incubation space to run their businesses and have access to research laboratories and libraries. Q Source: http://pib.nic.in/newsite/PrintRelease.aspx?relid=169569 66 Economically optimal or efficient allocation of goods and services is always reached in an economy when 1. Every resource in the economy is employed. 2. Resources go to those who need them the most. Which of the above is/are correct? A. 1 only B. 2 only C. Both 1 and 2 D. None Your Answer : C Correct Answer : D Answer Justification : Justification: Optimal allocation happens when goods and services are distributed according to consumer preferences and needs of corporations. Statement 1: It is possible that by employing all resources, we may reach a stage of disguised unemployment, for e.g. as common in rural areas. Statement 2: This is distribution based on equity and welfare concerns, which may not be efficient from an economic point of view. Q Source: Chapter 1: Ramesh Singh: Indian Economy 67 Consider the following statements. 1. The first demand for a Constituent Assembly was made right after the Non-cooperation movement was withdrawn. 2. The Constituent assembly was constituted in 1943 after the approval of the Cripps Proposals. Which of the above is/are correct? A. 1 only B. 2 only C. Both 1 and 2 D. None Your Answer : D Correct Answer : D Answer Justification : Justification: MN Roy had put forth the idea of a Constituent assembly of India in 1934. Later the INC demanded it. During the Second World War, this assertion for an independent Constituent Assembly formed only of Indians gained momentum and this was convened in December 1946. Between December 1946 and November 1949, the Constituent Assembly drafted a constitution for independent India. Q Source: Page 5: 8th NCERT Social and Political Life 68 Consider the following statements. The Multi Commodity Exchange of India Limited (MCX) 1. Is a listed exchange in India 2. operates under the regulatory framework of SEBI 3. does not offer trading in commodity futures contracts Select the correct answer using the codes below. A. 1 only B. 2 and 3 only C. 1 and 2 only D. 1, 2 and 3 Your Answer : C Correct Answer : C Answer Justification : Justification: Statement 1: It is India’s first listed exchange, is a state-of-the-art, commodity derivatives exchange that facilitates online trading. Statement 2: Most participants in the security market are under the purview of SEBI. Statement 3: MCX offers trading in varied commodity futures contracts across segments including bullion, industrial metals, energy and agricultural commodities. MCX is India’s leading commodity derivatives exchange with a market share of nearly 90 per cent in terms of the value of commodity futures contracts traded in Q1, FY 2017-18. Q Source: MCX has received SEBI’s approval to launch India’s first gold options contract 69 The Southernmost of these mountains ranges is A. Karakoram range B. Pir Panjal range C. Kaimur range D. Zanskar range Your Answer : D Correct Answer : C Answer Justification : Justification: Kaimur Range is the eastern portion of the Vindhya Range extending from around Katangi in Jabalpur district of Madhya Pradesh to around Sasaram in Rohtas district of Bihar. Q Source: Map based questions: India 70 The Economic Survey 2016-17 has proposed Transparency of Rules Act (TORA), a legislation to A. End any asymmetry of information regarding rules and regulations faced by an average citizen B. Improve law making by involving lay citizens in the process C. Instituting a Grievance Redressal Authority for each government service D. None of the above Your Answer : Correct Answer : A Answer Justification : Need: The ‘opaque mesh’ of regulations prevalent in India not only make life difficult for citizens who cannot feign ignorance of the rules as a valid defence, but also act as a magnet for corruption and endless litigation. Learning: The TORA is an attempt to change in some ways the relationship between the average normal citizen and the State. TORA will require all departments to mandatorily place all citizen-facing rules on their website. Officials will not be able to impose any rule not mentioned beforehand. All laws will have to be updated by the department while providing access to history of the same webpage. Once a department has shifted to the platform, it can be deemed “TORA compliant” and citizens can be sure that the information is authentic and updated. Q Source: http://www.insightsonindia.com/2017/08/12/insights-daily-current-affairs-12-aug-2017/ 71 It is said that Net Domestic Product (NDP) is a more realistic assessment of an economy’s resources than Gross Domestic Product (GDP). This is because 1. NDP takes into account taxes and subsidies paid which GDP does not. 2. NDP accounts for economic depreciation which GDP does not. Which of the above is/are correct? A. 1 only B. 2 only C. Both 1 and 2 D. None Your Answer : C Correct Answer : B Answer Justification : Justification: NDP is basically the GDP minus the total value of the ‘wear and tear’ (depreciation) that happened in the assets while the goods and services were being produced. So, if the depreciation levels are low, it may show the achievements of the economy in the area of research and development which have tried cutting the levels of depreciation in a historical time period. The governments of the economies decide and announce the rates by which assets depreciate (done in India by the Ministry of Commerce and Industry) and a list is published. Q Source: Chapter 1: Indian Economy: Ramesh Singh 72 E-Shakti for digitisation of Self Help Groups (SHGs) is a pilot project of A. Prime Minister’s Office (PMO) B. Micro Units Development & Refinance Agency Ltd. () C. Reserve Bank of India (RBI) D. National Bank for Agriculture and Rural Development (NABARD) Your Answer : D Correct Answer : D Answer Justification : Learning: It was initiated to address certain concerns like improving the quality of book keeping of SHGs and to enable banks to take informed credit decisions about the group through a Management Information System (MIS). The project aims at digitisation of all the SHG accounts to bring SHG members under the fold of Financial Inclusion. This increases bankers’ comfort in credit appraisal. Over 1 lakh SHGs have been digitised by now. Q Source: http://www.thehindubusinessline.com/economy/nabard-commences-work-on-digitising-shg-data/arti cle7447385.ece 73 Article 35A of the constitution was recently in news. This article concerns with #46460 A. Permanent residentship provision of the state of J&K B. Extraordinary powers of the President C. Relationship between the Union and the States D. Judicial review powers of the Supreme Court Your Answer : A Correct Answer : A Answer Justification : Learning: This is an issue intricately linked to the history of J&K. It would not be possible to describe the entire issue here. So, this is only a broad outline. Article 35A is a provision in the Constitution that empowers the Jammu and Kashmir legislature to define permanent residents of the state. An NGO’s petition contended that the State’s special autonomous status under Articles 35A and 370, was discriminatory against non-residents as far as government jobs and real estate purchases are concerned. You can read the issue at Insights Current events. Q Source: http://www.thehindu.com/news/national/plea-against-article-35a-may-be-heard-by-constitutional-ben ch-says-sc/article19490277.ece http://www.insightsonindia.com/2017/08/08/insights-daily-current-affairs-08-aug-2017/ 74 The NOTA voting option is NOT applicable in A. Lok Sabha elections B. Rajya Sabha elections C. Election of the President and Vice-President D. All of the above Your Answer : A Correct Answer : C Answer Justification : Learning: Option C: The option of NOTA would be against the dignity of the election of Presidential office. Option B: The Election Commission, in 2013, issued directions for providing the NOTA option in elections. But then, doubts were raised about its applicability in the Rajya Sabha polls. After examining the issue, the EC in 2014, directed that the option would also apply for elections to the Rajya Sabha. Recently, after a petition was filed, SC refused to stay that Election Commission circular issued in 2014 that introduced NOTA in the Rajya Sabha elections. Q Source: As mentioned above 75 The first city that was discovered in the Harappan civilization was A. Lothal B. Inamgaon C. Harappa D. Mohenjodaro Your Answer : C Correct Answer : C Answer Justification : Learning: In 1860s, when railway lines were being laid down for the first time in the Punjab, engineers stumbled upon the site of Harappa in present-day Pakistan. Then, in 1930s archaeologists found the site, and realised that this was one of the oldest cities in the subcontinent. As this was the first city to be discovered, all other sites from where similar buildings (and other things) were found were described as Harappan. These cities developed about 4700 years ago. Q Source: 6th NCERT History 76 It is one of Asia's two main opium-producing areas. It overlaps the mountains of three countries of Southeast Asia: Myanmar, Laos and Thailand. It is? A. Arakanyoma B. Golden triangle C. Tien Shan mountain system D. Extension of Tibetan Plateau Your Answer : Correct Answer : B Answer Justification : Learning: Along with Afghanistan in the Golden Crescent, it has been one of the most extensive opium-producing areas of Asia and of the world since the 1950s. The Golden Triangle designates the confluence of the Ruak River and the Mekong River, since the term has been appropriated by the Thai tourist industry to describe the nearby border tripoint of Thailand, Laos and Myanmar. Q Source: Map based questions: South Asia 77 Which of the following has the highest strength of members? A. Rajasthan Legislative assembly B. Rajya Sabha C. Lok Sabha D. Joint sitting of Uttar Pradesh Legislative assembly and Legislative Council Your Answer : C Correct Answer : C Answer Justification : Learning: Option C: Total strength of LS is 552 as set by the constitution. Option D: UP LA has a total of 403 members excluding one Anglo-Indian member who is nominated by the Governor. The Vidhan Parishad consists of 100 members. Joint strength is 503, which is lesser than LS. Option A: Rajasthan assembly has around 200 members. Q Source: Page 34: 8th NCERT Social and Political Life 78 RBI has announced the creation of a panel to consider creation of a Public Credit Registry (PCR) operated by the regulator. What is/are the advantages of PCR? 1. It can help banks in credit assessment and pricing of credit. 2. It will allow mandatory registration of all participatory notes (PN) FII entries in India. Which of the above is/are correct? A. 1 only B. 2 only C. Both 1 and 2 D. None Your Answer : Correct Answer : A Answer Justification : Justification: Statement 1: The PCR will be an extensive database of credit information for India that is accessible to all stakeholders. The idea is to capture all relevant information in one large database on the borrower and, in particular, the borrower’s entire set of borrowing contracts and outcomes. Further, it can help supervisors, regulators and banks in early intervention and effective restructuring of stressed bank credits. Q Source: http://www.insightsonindia.com/2017/08/03/insights-daily-current-affairs-03-aug-2017/ 79 In the context of Indus Valley civilization, which of these statements is the most appropriate? A. Indus valley people were fashion conscious. B. Indus Valley civilization was a closed economy. C. It was an atheist civilization. D. The civilization did not have access to urban amenities. Your Answer : A Correct Answer : A Answer Justification : Justification: Option A: Jewellery has been excavated. This tells us that Indus valley people were concerned with fashion. Option B: International trade was in practice in the civilization. This is because stone weights have been found which were used because there was a market where goods were traded. Option C: There is evidence of images of gods and goddesses like pashupati, who was considered the chief deity in the civilization. Option D: A number of urban amenities like bath structures, pukka houses, drainage system etc were present in these cities. Q Source: 6th NCERT History 80 Consider the following statements with reference to Indian polity. 1. Indian citizens are governed by laws and rules made by both Central and state governments at the same time. 2. The states are not merely agents of the Union government but draw their authority directly from the Constitution. Which of the above is/are correct? A. 1 only B. 2 only C. Both 1 and 2 D. None Your Answer : C Correct Answer : C Answer Justification : Justification: Constitutional existence of several tiers of government is the first pre-requisite of a federal polity. Under federalism, the states do not derive their authority from the Centre, but from the Constitution. While the centre may direct the states to perform certain actions, the fact is that they are an independent tier of government. The Constitution contains lists that detail the issues that each tier of government can make laws on. All persons in India are governed by laws and policies made by each of these levels of government. Q Source: Page 12: 8th NCERT Social and Political Life 81 Supreme Court (SC) is the final interpreter of the Constitution. But, the Parliament may disagree with the SC on the interpretation of the constitution. In such a case, which of these follows? A. Parliament declares the ruling of SC as unconstitutional. B. Parliament can amend the constitution to nullify the view of the judiciary. C. Parliament is bound to accept the views of the Judiciary, and cannot do anything in this matter. D. A Joint Parliamentary Committee can pass a resolution against the SC judgment declaring it as invalid. Your Answer : C Correct Answer : B Answer Justification : Justification: While SC is the final interpreter of the Constitution, parliament may disagree with the SC, and It is also empowered to amend the constitution in case of such disagreement. However, the SC gives the final word on the validity of such an amendment and can even strike it down. This tussle between the judiciary and Parliament over constitutional interpretation is best seen in the recent NJAC case or the historical A368 controversy. Q Source: Page 55: 8th NCERT Social and Political Life 82 The largest human source of carbon dioxide emissions is from A. Industrial manufacturing B. Combustion of fossil fuels C. Agricultural fields D. Clearing of forests Your Answer : A Correct Answer : B Answer Justification : Learning: Nearly 87 percent of all human-produced carbon dioxide emissions come from the burning of fossil fuels like coal, natural gas and oil. The remainder results from the clearing of forests and other land use changes (9%), as well as some industrial processes such as cement manufacturing (4%) Coal is the most carbon intensive fossil fuel. For every tonne of coal burned, approximately 2.5 tonnes of CO2e are produced Q Source: Based on the following: IISc researchers from Bengaluru have developed a highly sensitive, low cost nanometre-scale carbon monoxide (CO) sensor 83 The Mekong is a trans-boundary river in Southeast Asia. It does NOT pass through which of these countries? A. India B. China C. Vietnam D. Myanmar Your Answer : A Correct Answer : A Answer Justification : Learning: From the Tibetan Plateau the river runs through China's Yunnan Province, Myanmar, Laos, Thailand, Cambodia, and Vietnam. The MGC is an initiative by India and five ASEAN countries, Cambodia, Laos, Myanmar, Thailand and Vietnam for cooperation in tourism, culture, education, as well as transport and communications. It was launched in 2000 at Vientiane, Laos. Q Source: http://indianexpress.com/article/india/india-calls-for-expanding-cooperation-with-mgc-member-count ries-vk-singh-4787053/ 84 Friends of the Earth International (FoEI) 1. is an international network of environmental organizations 2. was founded as an anti-nuclear group 3. picks up causes of economic justice and supports human rights Select the correct answer using the codes below. A. 2 only B. 1 and 3 only C. 1 only D. 1, 2 and 3 Your Answer : Correct Answer : D Answer Justification : Justification: Operational in around seventy countries, Friends of the Earth was founded in 1969 as an anti-nuclear group. FOTE main mission was to lock up and prevent further development of nuclear energy. The current campaign priorities of Friends of the Earth internationally are: economic justice and resisting neoliberalism forests and biodiversity food sovereignty climate justice and energy (Including releasing the song "Love Song To the Earth") Q Source: https://www.theguardian.com/environment/2017/aug/14/shell-and-exxon-face-censure-over-claim-ga s-was-cleanest-fossil-fuel 85 Lithospheric plates move around very slowly – just a few millimetres each year. What is the reason behind this movement? A. Ocean currents B. Movement of magma inside earth C. Formation of folds on earth’s crust D. Rotation of earth Your Answer : D Correct Answer : B Answer Justification : Learning: The lithosphere is broken into a number of plates known as the Lithospheric plates. The molten magma inside the earth moves in a circular manner. These plates move because of the movement of the molten magma inside the earth. Lateral movements between lithospheric plates create transform faults at the sites of plate slippage. Q Source: 7th NCERT Geography 86 A Communist economy advocates 1. State ownership of all properties including labour in the economy 2. Absolute power to state in running and managing the economy Which of the above is/are correct? A. 1 only B. 2 only C. Both 1 and 2 D. None Your Answer : C Correct Answer : C Answer Justification : Justification: Communism is an economic system where the collective owns the factors of production. The four factors of production are labor, entrepreneurship, capital goods and natural resources. The following form major features of the communist economies: Abolition of property in land and application of all rents of land to public purposes. A heavy progressive or graduated income tax. Abolition of all right of inheritance. Q Source: Chapter 1: Ramesh Singh: Indian Economy 87 GOLF (Global Oscillations at Low Frequency) instrument, a joint project of ESA and NASA, studies the A. Internal structure of the sun B. Movements of underwater ocean currents C. Natural satellites of newly discovered exo-planets D. Black holes and supernovas Your Answer : Correct Answer : A Answer Justification : Learning: It will be on the Solar and Heliospheric Observatory (SOHO) spacecraft. The GOLF instrument studies the internal structure of the sun by measuring how the surface of the Sun oscillates. GOLF measures the velocity of the surface in the direction of sight - that is, how fast is the surface moving towards or away from the instrument. Q Source: http://economictimes.indiatimes.com/news/science/suns-core-rotates-four-times-faster-than-its-surfa ce/articleshow/59876214.cms 88 Annual Survey of Industries (ASI) is the principal source of industrial statistics in India. Consider the following with reference to it. 1. It does not cover unorganised or unregistered or informal sector enterprises. 2. It is conducted by Central Statistics Office (CSO) 3. Data from ASI comes with a lag of two years. Select the correct answer using the codes below. A. 1 and 2 only B. 2 and 3 only C. 3 only D. 1, 2 and 3 Your Answer : Correct Answer : D Answer Justification : Justification: Statement 1: For the other category of factories/establishments, which are not covered under the ASI, the information is collected through the unorganised sector surveys conducted by National Sample Survey Organisation (NSSO) every 5 years. Therefore, the ASI and the unorganised sector surveys together cover the complete manufacturing sector. Statement 3: Reference Year for ASI 2013-14 is the accounting year for the factory which is ending on FY 2014, while the survey would have been conducted in 2014-15. Q Source: Additional Research: Chapter 1: Indian Economy: Ramesh Singh 89 The Government has introduced ‘The Code on Wages’ Bill in the Lok Sabha that will consolidate which of the following laws? 1. Payment of Wages Act, 1936 2. Minimum Wages Act, 1998 3. Payment of Bonus Act, 1923 4. Equal Remuneration Act, 1919 Select the correct answer using the codes below. A. 1 only B. 2, 3 and 4 only C. 3 only D. 1, 2, 3 and 4 Your Answer : Correct Answer : A Answer Justification : Justification: Solve by elimination. If you notice, both 3 and 4 are written as pre-independence acts, which isn’t the case. It is actually - Payment of Bonus Act 1965 and Equal Remuneration Act 1976. Both progressive measures were passed post-independence and could not have been expected in a colonial setup. Statement 2: It was passed right after independence to ensure social justice, and the year of passing of this act should be common knowledge to you. It was “Minimum Wages Act 1948”. Learning: The bill seeks to amend the laws relating to wages and bonus which also seeks to empower the Centre to fix a “universal minimum wage” aimed to benefit over 40 crore unorganised sector workers. The bill seeks to amalgamate four laws — Payment of Wages Act 1936, Minimum Wages Act 1948, Payment of Bonus Act 1965 and Equal Remuneration Act 1976. The Code provides for the government to determine the minimum wages every five years using factors like skills required for the job, arduousness of work, geographical location of work place and other aspects. Q Source: http://www.insightsonindia.com/2017/08/11/insights-daily-current-affairs-11-aug-2017/ 90 Among the following, the South-east Asian region that is closest to equator is? A. Bangkok B. Singapore C. Manila D. Yangon Your Answer : Correct Answer : B Answer Justification : Justification: Q Source: Map based questions: South-east Asia 91 With reference to climate change financing, the principle of ‘common but differentiated responsibility’ evolved from the notion of 1. Nature as the common heritage of mankind 2. Equity in international law 3. Difference in economic and technical capacity of nations to tackle climate change 4. Historical differences in the contributions of developed and developing States in creating environmental problems Select the correct answer using the codes below. A. 1 and 4 only B. 1 and 2 only C. 2 and 3 only D. 1, 2, 3 and 4 Your Answer : D Correct Answer : D Answer Justification : Justification: Statement 2: Principle of equity implies fairness. Its implication can be seen in statement 4. Statement 4: For e.g. USA and China have contributed more to GHG emissions historically than LDCs like South Sudan or Zimbabwe. So, it is only fair that the larger share of responsibilities should go to USA and China. Statement 3: Moreover, not only responsibilities, but also capabilities are different. This implies that a larger burden cannot be given to LDCs or developed countries. So, the Rio Declaration states: “In view of the different contributions to global environmental degradation, States have common but differentiated responsibilities. Q Source: CBDR is a core principle of Kyoto Protocol and its 2nd phase was ratified by India recently 92 The Lok Sabha has passed the National Bank for Agriculture and Rural Development (Amendment) Bill, 2017. The bill provides for 1. Increase in the capital base of NABARD 2. Denationalization of NABARD 3. Delinking SHG finance from NABARD’s functions Select the correct answer using the codes below. A. 1 only B. 2 and 3 only C. 1 and 2 only D. 3 only Your Answer : Correct Answer : A Answer Justification : Justification: Statement 1: The Bill allows Union Government to increase capital of NABARD to Rs. Thirty thousand crore. Statement 2: The majority share will be held by the Union Government, and the rest by RBI. Statement 3: NABARD is responsible for providing and regulating facilities like credit for agricultural and industrial development in the rural areas. SHG microfinance is a core part of NABARD’s working, so 3 is wrong. There are other details as well, which you should read from Insights Current Events Q Source: As mentioned above 93 Eleventh and Twelfth schedule of the Constitution deal with A. Panchayat Raj institutions (PRIs) and Municipalities B. Land and tenure reforms C. Powers of the Union and the States D. Provisions for special administration of tribal areas Your Answer : A Correct Answer : A Answer Justification : Learning: Eleventh Schedule was added by 73rd amendment and has list of subjects under the Panchayat Raj institutions or rural local government, for e.g. A243G contains health, sanitation etc that can be taken care of by PRIs. It was added by 74th amendment and enlists the subjects under Municipalities or urban local government. The same goes for ULBs as well. Q Source: Additional Research: Page 101: 8th NCERT Social and Political Life 94 With reference to the governance of Indian forests, consider the following: 1. Indian Forest Act, 1927 empowers the government to declare any area to be a reserved forest or a protected forest. 2. Wild Life (Protection) Act, 1972 allows any area to be constituted as a "protected area". Which of the above is/are correct? A. 1 only B. 2 only C. Both 1 and 2 D. None Your Answer : C Correct Answer : C Answer Justification : Justification: India's forests are governed by two main laws, the Indian Forest Act, 1927 and the Wild Life (Protection) Act, 1972. The former allows declaration of an area to be a reserved forest, protected forest or village forest. The latter allows any area to be constituted as a national park, wildlife sanctuary, tiger reserve or community conservation area. However, this has often caused problems for the tribals, to resolve which a Forest Rights Act was constituted. We will cover this act in detail later. Q Source: Page 102: 8th NCERT Social and Political Life 95 Line joining which of these would give us the shortest route? A. Tirupati – Madurai B. Tirunelveli – Kochi C. Mysuru – Nizamabad D. Rajkot – Nanded Your Answer : Correct Answer : B Answer Justification : Q Source: Map based questions: India 96 Consider the following statements. 1. The chairman of the Council of States is elected directly by the Rajya Sabha. 2. The chairman of the Inter-State Council is nominated by the Union Cabinet. Which of the above is/are correct? A. 1 only B. 2 only C. Both 1 and 2 D. None Your Answer : C Correct Answer : D Answer Justification : Justification: Statement 1: She is not elected solely by the Rajya Sabha. She is elected jointly by LS and RS. In the case of Lok Sabha, the speaker is solely elected by the Lok Sabha. Statement 2: The Inter State Council composes of the following members: Prime Minister, Chairman. Chief Ministers of all states. Chief ministers of UTs with legislative assemblies. Q Source: Page 34: 8th NCERT Social and Political Life 97 Consider the following with regard to the Scheduled Castes and the Scheduled Tribes (Prevention of Atrocities) Act, 1989. 1. It provides for Special Courts for the trial of atrocities against SCs/STs. 2. It penalises anyone who wrongfully occupies or cultivates any land allotted to a member of a SC or a ST. 3. It applies to all the states of India. Select the correct answer using the codes below. A. 2 and 3 only B. 1 only C. 1 and 2 only D. 2 only Your Answer : Correct Answer : C Answer Justification : Justification: Statement 1: The Act recognizes that crimes against Dalit and tribal women are of a specific kind and provides for special courts. Statement 2: A land that belongs to a ST cannot be transferred to a member of other communities. So, any such person who does so shall be penalized. In cases where this has happened, the Constitution guarantees the right of tribal people to repossess their land. Statement 3: It does not apply to the state of J&K. Q Source: Additional Research: Page 102: 8th NCERT Social and Political Life 98 The credit of bringing the upper Ganges - Jamuna valley under large-scale cultivation goes to the Painted Grey Ware people. They could achieve this mainly because of 1. Their iron technology 2. The exclusion of resource intensive rice from the basket of crops Which of the above is/are correct? A. 1 only B. 2 only C. Both 1 and 2 D. None Your Answer : Correct Answer : A Answer Justification : Justification: Statement 1: They cleared the heavy jungle with the help of iron axes. Of the ploughshare no example has so far been found from this period, though the subsequent period has yielded examples. Statement 2: The excavations at Hastinapura brought to light the remains of rice. Q Source: http://en.unesco.org/silkroad/sites/silkroad/files/knowledge-bank-article/vol_I%20silk%20road_the% 20painted%20grey%20ware%20culture%20of%20the%20iron%20age.pdf 99 Consider the following statements about the following technologies in news. 1. CubeSats are miniature satellites that weigh few nanograms. 2. Film-Evaporation MEMS Tunable Array (FEMTA) uses pure water as the propellant for a spacecraft. Which of the above is/are correct? A. 1 only B. 2 only C. Both 1 and 2 D. None Your Answer : Correct Answer : B Answer Justification : Justification: Statement 1: CubeSats are basically miniature satellites which typically weigh around two kilograms. In the future, they have potential to carry out tasks like imaging and remotesensing currently performed by heavier satellites which are expensive to build and launch. Statement 2: The FEMTA system uses an innovative design of small thrusters that deliver bursts of water vapor to manoeuvre the spacecraft into different orbits. It uses pure water as the propellant since it is safe, green, easy to use and free from the risk of contaminating sensitive instruments by the backflow from plumes as in the case of thrusters using chemical propellants. Q Source: US Scientists develop micro-propulsion system based on liquid water 100 If a state government decides not to allow any labourers from other states to get public employment in its state, then it impinges on which of the following fundamental rights of an individual? A. Right to equality B. Right to freedom of movement C. Right to public employment D. This case is not a violation of fundamental rights. Your Answer : A Correct Answer : A Answer Justification : Justification: Option A: Right to equal opportunity for public employment is an important right under right to equality. It is certainly violated here because no socio-economic rationale can justify such restrictions on employment. Option B: Note than only employment (and not movement) has been restricted. Option C: There is no such right.